This page shows a recording of a live class. We're working hard to create our standard, concise explanation videos for the questions in this PrepTest. Thank you for your patience!

This is a sufficient assumption question, as indicated by the question stem: The conclusion above is properly drawn if which one of the following is assumed?

This is a very conditional-heavy stimulus.

Premise 1: /strike → increase wages

Premise 2: increase wages → sell sub

Conclusion: sell sub

Remember, conditionals have to be triggered in order for us to conclude anything from them. Since we’re trying to prove that Bell’s subsidiaries will be sold, we need something to trigger the first or second premise: either that there will not be a strike or that the wages will be increased.

Answer Choice (A) This does have anything to do with our premise or conclusion and it does help validate our conclusion. It doesn’t trigger our conditionals either.

Answer Choice (B) This is negating the necessary condition in the first premise, which would trigger backward, allowing us to conclude that the workers will do on strike. This answer choice also negates the sufficient condition in premise 2, which renders the rule useless. This is incorrect.

Correct Answer Choice (C) This triggers the first premise, which also triggers our second premise, leading to the conclusion that Bell’s subsidiaries will be sold.

Answer Choice (D) This AC is saying the president can increase wages; this does not mean that he will.

Answer Choice (E) This is additional information that does not clearly trigger our conditionals. The workers only wanting a wage increase does not help us validate the conclusion.


Comment on this

This is a strengthen question, and we know that because of the stem: Which one of the following, if true, most strengthens the argument?

Our stimulus tells us that asbestos (which they also include a nice little description of) poses health risks only if it’s disturbed and released into the environment. The author says that since removing it from buildings would disturb it, the government should not require the removal of all asbestos. This argument, as it stands, makes sense. You wouldn’t want to remove asbestos is disturbing it would cause harm. Our instinct is to be alarmed that anyone would support the idea of leaving asbestos in our walls and just living it. This argument assumes that the alternative (removing and putting it somewhere else) is worse.

Answer Choice (A) Plugging this back into the argument does not help the argument. We’re pointing out that asbestos is not as dangerous as all these other things, but that doesn’t strengthen the idea that the government should not require its removal.

Answer Choice (B) This is a conditional statement: if workers do not wear protective gear, asbestos can pose a health threat. What about if they do wear protective gear? Would there still be a threat? We don’t know! In other for this to interact with the stimulus, we would need to know that the workers are not wearing protective gear at the very least.

Answer Choice (C) These relative statements are meaningless: how much more? And is the less dangerous kind of asbestos dangerous in general? At what level? It could be that one is very dangerous, and the other is extremely dangerous. We’ve already said that asbestos, when disturbed, is dangerous - this relative statement does not do anything to strengthen the argument.

Answer Choice (D) This could potentially weaken the stimulus. They’re saying that since the asbestos will eventually get disturbed, what’s the point of preventing its removal now? This answer choice is essentially saying this restriction is unnecessary.

Answer Choice (E) This answer choice draws out a potential alternative proposal for removing asbestos and saying that it’s potentially dangerous: if we require that people remove all asbestos and then put it in a landfill (we can assume a landfill would be where opponents would want to dumb the asbestos), it’s still not safe.


1 comment

This page shows a recording of a live class. We're working hard to create our standard, concise explanation videos for the questions in this PrepTest. Thank you for your patience!

We know this is a sufficient assumption question because of the question stem: “which one… if assumed… would allow [the author] to draw her properly to draw her conclusion…” Interestingly, the conclusion is given to you in the stem. This should help in our formal analysis to figure out what the conclusion is.

Sufficient assumption questions tend to be very formal. We’re looking for a rule that would validate the conclusion, specifically by bridging the premise and conclusion through the rule. Not only are we extrapolating the rule from our argument, but we’re also using that rule to render the argument “valid.” The way to prephrase our answer choice is by tying our premises and conclusion together into a rule: “If [premise] → then [conclusion].”

Our first sentence tells us that an antitheft alarm may stop an attempted theft at night on a crowded street. I’m imagining an incredibly loud siren going off in a middle of a downtown area... I think it’s possible that theft could be prevented with a loud alarm like that.

The next sentence gives an alternative cause for the alarm going off: instead of a thief, it could be a branch or another form of contact.

The third sentence puts aside the causes and says in any of the situations in which the alarm goes off at night, it’s disturbing the sleep of people in the neighborhood. Makes sense - a blaring alarm would definitely disturb my sleep!

Our last sentence, the conclusion which we read in our stem, is: out of consideration for our neighbors, we should deactivate the alarms when parking in crowded city neighborhoods at night.

Why should they?? Protecting a car is probably more important and valuable to its owner than someone else’s sleep, and there doesn’t seem to be a moral code that says people should deactivate their cars if it bothers people’s sleep. On the chance that the car is getting stolen, an alarm would be the best thing to help avoid losing the car! This prescriptive conclusion is assuming that others’ sleep is more important than protecting someone’s car.

What we need is a rule that bridges our premises to our conclusion and validates this prescription. Our premise here is that if the alarm goes off, then people’s sleep is disturbed, and we should be considerate of this. Our conclusion is that owners of these cars should deactivate their alarms. Put them together to make our rule! If people’s sleep is getting disturbed and we should be considerate of this → owners should deactivate car alarms at night.

Answer Choice (A) This would weaken the argument! This is putting the protection of peoples’ cars over peoples’ sleep.

Answer Choice (B) In most cases? What about some cases in which it’s actually a theft? And besides this, the answer choice still does not justify why we should deactivate our alarms. Just because it is a false alarm, it does not validate the argument.

Correct Answer Choice (C) While it’s not a perfect paraphrase of our rule, it gets to the idea that sleep is more important than protecting cars.

Answer Choice (D) Remember, our conclusion is prescriptive. How does saying “people who have alarms are inconsiderate” help our argument? Does this mean they should deactivate their alarms? No – this is out.

Answer Choice (E) We don’t really care about what happens to the alarms during the day, we’re concerned with why people should turn alarms off at night. This is supplemental information that does nothing for our stimulus.


Comment on this

This page shows a recording of a live class. We're working hard to create our standard, concise explanation videos for the questions in this PrepTest. Thank you for your patience!

Here we have a five star question with a peculiar question stem. We should at least know that this is a strengthening question because it asks: Which one of the following ethical criteria, if valid, would serve to support the journalist’s conclusion. The question adds another requirement of a correct answer, stating it will support the conclusion “while placing the least constraint on the flow of reported information.” An extra qualification can throw you off, but thankfully this isn’t something you see on LR questions anymore. The LSAT writer wants this requirement to lurk in the back of your mind and distract you, but when an extra requirement is added, you should focus on approaching the question as normal and only bring in the second requirement if you have multiple answers which meet the first.

The stimulus tells us about a journalist in a civil war who found evidence that the government was responsible for refugees starving. Unfortunately, government censors removed any mention of the government from the journalist’s report. The journalist concluded (alarm bells should be going off for you here) that it was ethically permissible to file the censored report, because (more alarm bells) it would be preceded by a notice that it had gone through government censors. This is a rare case where the stimulus is more straightforward than the question stem.

The key sentence is the last one, it gives us the conclusion which a correct answer must support, and the journalist’s reasoning for it. She believes that although the report had been altered by the government, it is still ethically permissible to publish the censored version if readers are explicitly told it had been cleared by government censors. The correct answer will give us an ethical principle which support’s the journalist’s reasoning, while incorrect answers will fail to do this or even outright contradict the journalist’s conclusion. Let’s move to the answers now:

Answer Choice (A) The first thing we should do when judging this answer is ignore “it is ethical in general to report known facts”, because it is a broad statement which adds very little to justify the journalist’s decision. What we are interested in is the rest of the answer, which only tells us what is unethical. The “if” should jump at you, and if we convert this answer to a conditional we get the principle that “if the omitted facts of a report might substantially alter an impression of the subject which would be congruent with the included facts, then it is unethical.” Congruent is thrown in to try and confuse you, but essentially just means consistent, i.e non-contradictory. This answer is incorrect, it greatly weakens our journalist’s conclusion because she did omit facts that would alter a reader’s impression of the government. I think it is safe to say that if you found out a government was starving people, your impression of it would be altered.

Answer Choice (B) This answer also begins with the general principle “it is ethical to report known facts” which does nothing to help us and should be ignored. Like A the rest of the answer only tells us the conditions for being unethical. This answer weakens what we wish to support, because it is true that the journalist didn’t report facts she knew which might exonerate other factors (like the rebels and nature) by assigning responsibility for the refugees starvation to the government.

Answer Choice (C) If this were a weakening question, this would be a great answer because it necessarily excludes the journalist’s action from being ethical, seeing as she filed a report which a censor had deleted unfavorable material from. But this is a strengthening question, and therefore C is incorrect.

Correct Answer Choice (D) This answer includes an if statement within an unless statement. If we translate it, we can get “If there is a censorship warning, then it is not true that if you report censored material, it is unethical.” This supports the journalist’s conclusion by supporting her reasoning that a censorship notice negates any ethical issues with publishing censored material.

Answer Choice (E) You know what to omit here. This answer is mostly a repeat of D, but with the addition of an AND statement. To be ethical a censored report’s reported facts (i.e the ones that weren’t censored) must now also not give a misleading impression by themselves. This answer is essentially a combination of A and D, and is incorrect for the same reason that A is. The reported facts by themselves, independent from the censored facts and the censorship notice, do give a misleading impression of the government. If you just read the facts that were included, you wouldn’t get the impression that the government was starving people.


Comment on this

This is a Method of Reasoning question, and we know this because of the question stem: “Judith’s reply to Anthony’s argument relies on which one of the following argumentative strategies?”

Anthony claims that using marijuana “definitely leads,” or causes, people to use heroin. His evidence is that the two tend to happen together, meaning they are correlated. He is jumping from correlation in his premise to causation in his conclusion. Our girl, Judith totally picks up on this. She’s saying this could be true, the statistics, which is his premise/evidence, does not support the causal conclusion. She then cites another correlation between water consumption and heroin use, which would lead to a berserk argument: based on Anthony’s argument for things being correlated with heroin use causes heroin use, water consumption causes heroin use. Here, she’s employing an analogy to reveal how the argument is flawed. She concludes that Anthony’s conclusion isn’t necessarily supported by his evidence/premise. Her evidence/premise is an analogy she puts forward to point out the issue in his reasoning.

Answer Choice (A) Judith never contests the factual accuracy of the statistics that Anthony offers. They could be completely accurate, but they do not support his causal conclusion.

Answer Choice (B) Judith is not undermining the credibility of his conclusion - if her first sentence, she even concedes that Anthony’s conclusion could be true. She specifically takes issue with the way he supports his conclusion.

Answer Choice (C) The example Judith cites is not promoting heroin usage; she is saying that drawing a correlation between X thing and heroin and then saying thing X causes or promotes heroin use is not true.

Correct Answer Choice (D) Judith is not questioning the premise or the conclusion, she’s simply questioning the support, or rather, the line of reasoning, between the two. By putting forward that analogy, she is showing how his line of reasoning is flawed.

Answer Choice (E) This is not good - the possibility of ever establishing a causal connection? That’s too extreme.


Comment on this

This page shows a recording of a live class. We're working hard to create our standard, concise explanation videos for the questions in this PrepTest. Thank you for your patience!

3 comments

We can identify this question as Method of Reasoning because of the question stem: “T responds to S’s argument by…” When dealing with a Method of Reasoning question, we know we are looking for an answer choice that correctly describes the structure of our entire argument. Our correct answer is going to fit the argument exactly. Our wrong answer choices likely explain argument structures we are familiar with, but that simply don’t apply to the specific question we are looking at. Knowing what the right and wrong answers are going to do, we can jump into the stimulus.

This question presents us with two speakers. Right away, we should recognize that there are two conclusions and two reasons behind them. In this question we are presented with two speakers who take varying positions on the use of voting as a comparison to enlisting in the military. First, speaker S presents a belief that people who are old enough to fight in wars are old enough to vote. On the basis that their government enlists 17 year olds to fight, speaker S ultimately concludes the group should be allowed to vote. This argument is immediately questionable. What does knowing how to fight in a war have to do with having the skills to vote? Speaker S assumes that the skills applicable to war transfer to the skills necessary to vote.

Our second speaker begins by pointing out the assumption in speaker S’s argument. Speaker T tells us that so long as we go along with the assumption - voting and war require the same skills - the first speaker would make complete sense. But T points out that assumption is not reality presenting the different skills required between the two activities - physical strength for combat, and reasoning power required to vote.

Knowing we are looking for the answer choice pointing out the assumption T exemplifies, we can proceed into answer choice elimination.

Answer Choice (A) This answer choice does not line up with what we see in the stimulus. To start off, this answer says T points out evidence that is good for S’s conclusion. But we know that T is actually pointing out something bad by explaining how S’s assumption about the transfer of skills from war to voting does not make sense.

Answer Choice (B) We can get rid of this answer choice based on understanding of rights. T is not questioning the opponent’s competence. For this answer choice to be correct, we would need to see some sort of direct reference to S’s knowledge rather than information that questions the validity of the overall assumption.

Answer Choice (C) This answer choice do4es not line up with what we predicted from the stimulus. Aside from the fact that the issue of obligation does not appear in our original stimulus, speaker T does not base their argument on something that S has ignored.

Correct Answer Choice (D) This is exactly what we are looking for! This answer choice correctly summarizes the structure of our entire argument by explaining how the second speaker calls into question the assumptions of speaker S.

Answer Choice (E) We can eliminate this answer choice once we get to the word opposite. In saying our argument argues for a conclusion opposite to the one drawn, we would need to see speaker T conclude that 17 year olds should not have the right to vote. Speaker T does not go quite that far. Instead of saying a 17 year old should not have the right to vote, our second speaker merely states that speaker S’s reasoning does not make sense.


1 comment

Which one of the following is most strongly supported by the information above?

This is a Most Strongly Supported question.

Superstring theory is a controversial new theory in physics that purports, unlike more established physical theories, to explain the nature and existence of gravity.

Have you heard of the controversial new theory known as superstring theory? What’s so controversial about it, you ask? Get this – it tries to explain the nature and existence of gravity. Yeah, I know – this theory’s asking to be made a fool of. All the more established physical theories know they can’t explain gravity, so they don’t front as if they do. (This last claim about what more established physical theories do is hiding in the author’s use of “unlike” – that means the more established physical theories don’t do what we’re told superstring theory does.)

A major problem with superstring theory is that to test it we would have to build a particle accelerator 100 trillion kilometers long.

Let’s break down what makes superstring theory so naive. First up is how difficult it would be to test what superstring theory is saying about gravity. We’d have to build an impossibly long particle accelerator – your mind can’t even grasp how long this accelerator needs to be. So superstring theory is saying stuff that we can’t test – that’s strike one.

Another problem is that superstring theory has had no success in adequately explaining why the force of gravity is not stronger or weaker than it is.

The next problem is superstring theory has completely failed in explaining why gravity isn’t stronger or weaker than it is. Here’s what I mean. If you jump from a plane, you’ll fall to Earth at a rate of about 120 miles per hour according to Google. But why don’t you fall faster, like 1 million miles per hour? Or 1 mile per hour? Why do you fall at 120 miles per hour instead of some other speed? Why is gravity as powerful as it is, but not more or less? Superstring theory doesn’t adequately explain this. That’s strike two.

Is there a third strike? Maybe, not sure. That’s as far as the stimulus cares to slam superstring theory.

Answer Choice (A) Superstring theory would be more successful if superstring theorists attempted to explain why the force of gravity is not stronger or weaker than it is.

This is a tempting answer, because we do know that superstring’s inability to provide an adequate explanation of why gravity is the way it is counts as one of the strikes against it. But the problem is the strike against superstring theory is for having an inadequate explanation. We’re not criticizing superstring theory for failing to attempt an explanation. We want an adequate explanation – not an attempt that fails.

This answer would have been better if it had said, “Superstring theory would be more successful if it adequately explained why the force of gravity is not stronger or weaker than it is.”

Answer Choice (B) Physical theories that are better established than superstring theory provide better explanations of physical phenomena than does superstring theory.

Be careful about reading too much into the stimulus’s claims about more established theories. Remember, we know that more established theories don’t purport to explain the nature and existence of gravity. This is what made superstring theory controversial – it pretends like it can explain gravity. The more established theories don’t even try to explain gravity. But beyond that, we don’t know anything else about the more established physical theories. Do they give better explanations of “physical phenomena”? Maybe, maybe not. Perhaps superstring theory is actually better at explaining the movement of water, or wind, or temperature, or other physical aspects of the world.

Answer Choice (C) Some physical theory more established than superstring theory has had at least some success in explaining why the force of gravity is not stronger or weaker than it is.

This answer tries to tempt you by using the phrase “some physical theory” and “at least some success.” This kind of weak language is, all else equal, attractive on a Most Strongly Supported question, because it’s easier to prove a statement about “some” things (at least one) than it is about “most” (over half) or “all” (100%).

But despite this weak language, (C) still has no support from the stimulus. We don’t know how successful any other theory is at explaining any aspect of gravity.

In fact, the first sentence establishes that physical theories that are more established than superstring theory do not purport to explain the nature and existence of gravity. They don’t even try to do it.

While this doesn’t, by itself, prove that those theories can’t have success at explaining gravity (because perhaps a theory might successfully explain something that it never purported to explain), it does mean that we have no basis on which to speculate about those theories and their success. If they don’t even try to explain gravity, what reason do we have to believe that one of those theories has some success at an explanation? Simply because they’re more established? But those theories could be more established because they’ve been around longer, or because they are easier to understand, or for any number of reasons that have nothing to do with success at explaining gravity.

Answer Choice (D) A physical theory cannot be true if testing that theory would require us to build a particle accelerator 100 trillion kilometers long.

This goes too far – if it had said that a physical theory has a problem if testing it would require the long accelerator, then it would be supported. But we can’t speak to whether the theory is true or false. We can’t test superstring theory, which is one of its problems. But it might still be correct. We just wouldn’t know whether it’s correct, because we can’t test it.

Correct Answer Choice (E) A theory that purports to explain the nature of a force is deficient if it cannot account for the strength of that force.

If (E) said the following, it would be much easier to select:

“A theory that purports to explain the nature of a force has a problem if it cannot explain why that force is not stronger or weaker than it is.”

This would be supported by the last sentence, which states that superstring theory has a problem because it can’t explain why gravity isn’t stronger or weaker than it is.

But, as you know, the LSAT often speaks about the same concept using different words and phrases. In (E), the LSAT uses the word “deficient” to refer to the concept of a theory’s having a “problem.” And it’s using the phrase “account for the strength” to refer to the idea of explaining why gravity is not stronger or weaker than it is.

Both of these changes are fair. A theory that has a problem cannot be perfect – it’s lacking something. If it weren’t lacking anything, then it wouldn’t make sense to say that the theory has a problem. And a theory that is lacking something is deficient.

The word “account” can mean “explain.” So you can read “... if it cannot account for the strength of that force” as “... if it cannot explain the strength of that force.” This is a slightly more general way of describing the inability to explain why gravity isn’t stronger or weaker than it is. Is it a perfect match? I’m not sure. There might be some difference between explaining why a force isn’t stronger or weaker than it is and “explaining the strength of that force.” But the concepts are close enough for a Most Strongly Supported question, and there is no other answer that has more support.


4 comments